Direct passage from $n$ prime to $n$ non-prime in Euler Totient Function $Phi$












0












$begingroup$


I wish to derive, for a proof, the Euler Totient Function starting from the case $n$ prime to $n$ non-prime.



Let $n$ prime, we know $Phi(n)=n-1$. But what if now I assume $n=p_0^{a_0}p_1^{a_1}...p_r^{a_r}$ and I want to reach $Phi(n)=n prod_{p|n}^{}(1-1/p) $ from there? Like:



$Phi(p_0^{a_0}p_1^{a_1}...p_r^{a_r})=p_0^{a_0}p_1^{a_1}...p_r^{a_r}-1 Rightarrow p_0^{a_0}p_1^{a_1}...p_r^{a_r}(1-1/(p_0^{a_0}p_1^{a_1}...p_r^{a_r}))=?$



Is this kind of approach possible for this formula?










share|cite|improve this question









$endgroup$








  • 3




    $begingroup$
    The two steps are $phi(p^k) = p^k-p^{k-1}$ and $phi(nm) = phi(n) phi(m)$ whenever $gcd(n,m)=1$ thus $phi(n) = prod_{p^k |n} phi(p^k) = nprod_{p^k |n} frac{phi(p^k) }{p^k}= n prod_{p| n} (1-p^{-1})$
    $endgroup$
    – reuns
    Jan 15 at 8:19








  • 1




    $begingroup$
    In particular, it is not true that $phi(p_0^{a_0}p_1^{a_1}cdots p_r^{a_r})=p_0^{a_0}p_1^{a_1}cdots p_r^{a_r}-1$ when $p_0^{a_0}p_1^{a_1}cdots p_r^{a_r}$ is not prime.
    $endgroup$
    – Greg Martin
    Jan 15 at 8:28










  • $begingroup$
    So such kind of derivation, as I suppose, it's not possible. Thanks
    $endgroup$
    – Alessar
    Jan 15 at 8:31










  • $begingroup$
    @reuns the symbol "$||$" means for "that divide"?
    $endgroup$
    – Alessar
    Jan 15 at 9:13






  • 1




    $begingroup$
    $p^k| n$ iff $p^k | n$ and $p^{k+1} nmid n$. In this setting $p$ means (ranging over) the primes and $p^k$ means over the prime powers
    $endgroup$
    – reuns
    Jan 15 at 9:17


















0












$begingroup$


I wish to derive, for a proof, the Euler Totient Function starting from the case $n$ prime to $n$ non-prime.



Let $n$ prime, we know $Phi(n)=n-1$. But what if now I assume $n=p_0^{a_0}p_1^{a_1}...p_r^{a_r}$ and I want to reach $Phi(n)=n prod_{p|n}^{}(1-1/p) $ from there? Like:



$Phi(p_0^{a_0}p_1^{a_1}...p_r^{a_r})=p_0^{a_0}p_1^{a_1}...p_r^{a_r}-1 Rightarrow p_0^{a_0}p_1^{a_1}...p_r^{a_r}(1-1/(p_0^{a_0}p_1^{a_1}...p_r^{a_r}))=?$



Is this kind of approach possible for this formula?










share|cite|improve this question









$endgroup$








  • 3




    $begingroup$
    The two steps are $phi(p^k) = p^k-p^{k-1}$ and $phi(nm) = phi(n) phi(m)$ whenever $gcd(n,m)=1$ thus $phi(n) = prod_{p^k |n} phi(p^k) = nprod_{p^k |n} frac{phi(p^k) }{p^k}= n prod_{p| n} (1-p^{-1})$
    $endgroup$
    – reuns
    Jan 15 at 8:19








  • 1




    $begingroup$
    In particular, it is not true that $phi(p_0^{a_0}p_1^{a_1}cdots p_r^{a_r})=p_0^{a_0}p_1^{a_1}cdots p_r^{a_r}-1$ when $p_0^{a_0}p_1^{a_1}cdots p_r^{a_r}$ is not prime.
    $endgroup$
    – Greg Martin
    Jan 15 at 8:28










  • $begingroup$
    So such kind of derivation, as I suppose, it's not possible. Thanks
    $endgroup$
    – Alessar
    Jan 15 at 8:31










  • $begingroup$
    @reuns the symbol "$||$" means for "that divide"?
    $endgroup$
    – Alessar
    Jan 15 at 9:13






  • 1




    $begingroup$
    $p^k| n$ iff $p^k | n$ and $p^{k+1} nmid n$. In this setting $p$ means (ranging over) the primes and $p^k$ means over the prime powers
    $endgroup$
    – reuns
    Jan 15 at 9:17
















0












0








0





$begingroup$


I wish to derive, for a proof, the Euler Totient Function starting from the case $n$ prime to $n$ non-prime.



Let $n$ prime, we know $Phi(n)=n-1$. But what if now I assume $n=p_0^{a_0}p_1^{a_1}...p_r^{a_r}$ and I want to reach $Phi(n)=n prod_{p|n}^{}(1-1/p) $ from there? Like:



$Phi(p_0^{a_0}p_1^{a_1}...p_r^{a_r})=p_0^{a_0}p_1^{a_1}...p_r^{a_r}-1 Rightarrow p_0^{a_0}p_1^{a_1}...p_r^{a_r}(1-1/(p_0^{a_0}p_1^{a_1}...p_r^{a_r}))=?$



Is this kind of approach possible for this formula?










share|cite|improve this question









$endgroup$




I wish to derive, for a proof, the Euler Totient Function starting from the case $n$ prime to $n$ non-prime.



Let $n$ prime, we know $Phi(n)=n-1$. But what if now I assume $n=p_0^{a_0}p_1^{a_1}...p_r^{a_r}$ and I want to reach $Phi(n)=n prod_{p|n}^{}(1-1/p) $ from there? Like:



$Phi(p_0^{a_0}p_1^{a_1}...p_r^{a_r})=p_0^{a_0}p_1^{a_1}...p_r^{a_r}-1 Rightarrow p_0^{a_0}p_1^{a_1}...p_r^{a_r}(1-1/(p_0^{a_0}p_1^{a_1}...p_r^{a_r}))=?$



Is this kind of approach possible for this formula?







abstract-algebra elementary-number-theory modular-arithmetic totient-function






share|cite|improve this question













share|cite|improve this question











share|cite|improve this question




share|cite|improve this question










asked Jan 15 at 8:17









AlessarAlessar

29615




29615








  • 3




    $begingroup$
    The two steps are $phi(p^k) = p^k-p^{k-1}$ and $phi(nm) = phi(n) phi(m)$ whenever $gcd(n,m)=1$ thus $phi(n) = prod_{p^k |n} phi(p^k) = nprod_{p^k |n} frac{phi(p^k) }{p^k}= n prod_{p| n} (1-p^{-1})$
    $endgroup$
    – reuns
    Jan 15 at 8:19








  • 1




    $begingroup$
    In particular, it is not true that $phi(p_0^{a_0}p_1^{a_1}cdots p_r^{a_r})=p_0^{a_0}p_1^{a_1}cdots p_r^{a_r}-1$ when $p_0^{a_0}p_1^{a_1}cdots p_r^{a_r}$ is not prime.
    $endgroup$
    – Greg Martin
    Jan 15 at 8:28










  • $begingroup$
    So such kind of derivation, as I suppose, it's not possible. Thanks
    $endgroup$
    – Alessar
    Jan 15 at 8:31










  • $begingroup$
    @reuns the symbol "$||$" means for "that divide"?
    $endgroup$
    – Alessar
    Jan 15 at 9:13






  • 1




    $begingroup$
    $p^k| n$ iff $p^k | n$ and $p^{k+1} nmid n$. In this setting $p$ means (ranging over) the primes and $p^k$ means over the prime powers
    $endgroup$
    – reuns
    Jan 15 at 9:17
















  • 3




    $begingroup$
    The two steps are $phi(p^k) = p^k-p^{k-1}$ and $phi(nm) = phi(n) phi(m)$ whenever $gcd(n,m)=1$ thus $phi(n) = prod_{p^k |n} phi(p^k) = nprod_{p^k |n} frac{phi(p^k) }{p^k}= n prod_{p| n} (1-p^{-1})$
    $endgroup$
    – reuns
    Jan 15 at 8:19








  • 1




    $begingroup$
    In particular, it is not true that $phi(p_0^{a_0}p_1^{a_1}cdots p_r^{a_r})=p_0^{a_0}p_1^{a_1}cdots p_r^{a_r}-1$ when $p_0^{a_0}p_1^{a_1}cdots p_r^{a_r}$ is not prime.
    $endgroup$
    – Greg Martin
    Jan 15 at 8:28










  • $begingroup$
    So such kind of derivation, as I suppose, it's not possible. Thanks
    $endgroup$
    – Alessar
    Jan 15 at 8:31










  • $begingroup$
    @reuns the symbol "$||$" means for "that divide"?
    $endgroup$
    – Alessar
    Jan 15 at 9:13






  • 1




    $begingroup$
    $p^k| n$ iff $p^k | n$ and $p^{k+1} nmid n$. In this setting $p$ means (ranging over) the primes and $p^k$ means over the prime powers
    $endgroup$
    – reuns
    Jan 15 at 9:17










3




3




$begingroup$
The two steps are $phi(p^k) = p^k-p^{k-1}$ and $phi(nm) = phi(n) phi(m)$ whenever $gcd(n,m)=1$ thus $phi(n) = prod_{p^k |n} phi(p^k) = nprod_{p^k |n} frac{phi(p^k) }{p^k}= n prod_{p| n} (1-p^{-1})$
$endgroup$
– reuns
Jan 15 at 8:19






$begingroup$
The two steps are $phi(p^k) = p^k-p^{k-1}$ and $phi(nm) = phi(n) phi(m)$ whenever $gcd(n,m)=1$ thus $phi(n) = prod_{p^k |n} phi(p^k) = nprod_{p^k |n} frac{phi(p^k) }{p^k}= n prod_{p| n} (1-p^{-1})$
$endgroup$
– reuns
Jan 15 at 8:19






1




1




$begingroup$
In particular, it is not true that $phi(p_0^{a_0}p_1^{a_1}cdots p_r^{a_r})=p_0^{a_0}p_1^{a_1}cdots p_r^{a_r}-1$ when $p_0^{a_0}p_1^{a_1}cdots p_r^{a_r}$ is not prime.
$endgroup$
– Greg Martin
Jan 15 at 8:28




$begingroup$
In particular, it is not true that $phi(p_0^{a_0}p_1^{a_1}cdots p_r^{a_r})=p_0^{a_0}p_1^{a_1}cdots p_r^{a_r}-1$ when $p_0^{a_0}p_1^{a_1}cdots p_r^{a_r}$ is not prime.
$endgroup$
– Greg Martin
Jan 15 at 8:28












$begingroup$
So such kind of derivation, as I suppose, it's not possible. Thanks
$endgroup$
– Alessar
Jan 15 at 8:31




$begingroup$
So such kind of derivation, as I suppose, it's not possible. Thanks
$endgroup$
– Alessar
Jan 15 at 8:31












$begingroup$
@reuns the symbol "$||$" means for "that divide"?
$endgroup$
– Alessar
Jan 15 at 9:13




$begingroup$
@reuns the symbol "$||$" means for "that divide"?
$endgroup$
– Alessar
Jan 15 at 9:13




1




1




$begingroup$
$p^k| n$ iff $p^k | n$ and $p^{k+1} nmid n$. In this setting $p$ means (ranging over) the primes and $p^k$ means over the prime powers
$endgroup$
– reuns
Jan 15 at 9:17






$begingroup$
$p^k| n$ iff $p^k | n$ and $p^{k+1} nmid n$. In this setting $p$ means (ranging over) the primes and $p^k$ means over the prime powers
$endgroup$
– reuns
Jan 15 at 9:17












1 Answer
1






active

oldest

votes


















1












$begingroup$

To be precise, your "kind" of derivation is not possible. This is because $phi(p_0^{a_0}p_1^{a_1}...p_r^{a_r})neq p_0^{a_0}p_1^{a_1}...p_r^{a_r}-1$.



Think about what the totient function entails: the number of integers less than the input that are coprime with it. Only with prime inputs is this one less than the input.



The totient function, however, does retain two properties, namely as reuns mentioned, that $phi(p^k)=p^k-p^{k-1}$ and $phi(n)phi(m)=phi(mn)$ when $m$ and $n$ are coprime. The proof of these is easily verifiable given a google search.



Thus, we can have a "direct" approach. Since powers of primes are always coprime to one another,
begin{align*}
phi(n)&=phi(p_0^{a_0}p_1^{a_1}...p_r^{a_r})\
&=phi(p_0^{a_0})phi(p_1^{a_1})...phi(p_r^{a_r})\
&=(p_0^{a_0}-p_0^{a_0-1})(p_1^{a_1}-p_1^{a_1-1})...(p_r^{a_r}-p_r^{a_r-1})\
&=p_0^{a_0}(1-1/p_0)p_1^{a_1}(1-1/p_1)...p_r^{a_r}(1-1/p_r)\
&=ndisplaystyleprod_{p|n}(1-1/p).
end{align*}






share|cite|improve this answer









$endgroup$













    Your Answer





    StackExchange.ifUsing("editor", function () {
    return StackExchange.using("mathjaxEditing", function () {
    StackExchange.MarkdownEditor.creationCallbacks.add(function (editor, postfix) {
    StackExchange.mathjaxEditing.prepareWmdForMathJax(editor, postfix, [["$", "$"], ["\\(","\\)"]]);
    });
    });
    }, "mathjax-editing");

    StackExchange.ready(function() {
    var channelOptions = {
    tags: "".split(" "),
    id: "69"
    };
    initTagRenderer("".split(" "), "".split(" "), channelOptions);

    StackExchange.using("externalEditor", function() {
    // Have to fire editor after snippets, if snippets enabled
    if (StackExchange.settings.snippets.snippetsEnabled) {
    StackExchange.using("snippets", function() {
    createEditor();
    });
    }
    else {
    createEditor();
    }
    });

    function createEditor() {
    StackExchange.prepareEditor({
    heartbeatType: 'answer',
    autoActivateHeartbeat: false,
    convertImagesToLinks: true,
    noModals: true,
    showLowRepImageUploadWarning: true,
    reputationToPostImages: 10,
    bindNavPrevention: true,
    postfix: "",
    imageUploader: {
    brandingHtml: "Powered by u003ca class="icon-imgur-white" href="https://imgur.com/"u003eu003c/au003e",
    contentPolicyHtml: "User contributions licensed under u003ca href="https://creativecommons.org/licenses/by-sa/3.0/"u003ecc by-sa 3.0 with attribution requiredu003c/au003e u003ca href="https://stackoverflow.com/legal/content-policy"u003e(content policy)u003c/au003e",
    allowUrls: true
    },
    noCode: true, onDemand: true,
    discardSelector: ".discard-answer"
    ,immediatelyShowMarkdownHelp:true
    });


    }
    });














    draft saved

    draft discarded


















    StackExchange.ready(
    function () {
    StackExchange.openid.initPostLogin('.new-post-login', 'https%3a%2f%2fmath.stackexchange.com%2fquestions%2f3074184%2fdirect-passage-from-n-prime-to-n-non-prime-in-euler-totient-function-phi%23new-answer', 'question_page');
    }
    );

    Post as a guest















    Required, but never shown

























    1 Answer
    1






    active

    oldest

    votes








    1 Answer
    1






    active

    oldest

    votes









    active

    oldest

    votes






    active

    oldest

    votes









    1












    $begingroup$

    To be precise, your "kind" of derivation is not possible. This is because $phi(p_0^{a_0}p_1^{a_1}...p_r^{a_r})neq p_0^{a_0}p_1^{a_1}...p_r^{a_r}-1$.



    Think about what the totient function entails: the number of integers less than the input that are coprime with it. Only with prime inputs is this one less than the input.



    The totient function, however, does retain two properties, namely as reuns mentioned, that $phi(p^k)=p^k-p^{k-1}$ and $phi(n)phi(m)=phi(mn)$ when $m$ and $n$ are coprime. The proof of these is easily verifiable given a google search.



    Thus, we can have a "direct" approach. Since powers of primes are always coprime to one another,
    begin{align*}
    phi(n)&=phi(p_0^{a_0}p_1^{a_1}...p_r^{a_r})\
    &=phi(p_0^{a_0})phi(p_1^{a_1})...phi(p_r^{a_r})\
    &=(p_0^{a_0}-p_0^{a_0-1})(p_1^{a_1}-p_1^{a_1-1})...(p_r^{a_r}-p_r^{a_r-1})\
    &=p_0^{a_0}(1-1/p_0)p_1^{a_1}(1-1/p_1)...p_r^{a_r}(1-1/p_r)\
    &=ndisplaystyleprod_{p|n}(1-1/p).
    end{align*}






    share|cite|improve this answer









    $endgroup$


















      1












      $begingroup$

      To be precise, your "kind" of derivation is not possible. This is because $phi(p_0^{a_0}p_1^{a_1}...p_r^{a_r})neq p_0^{a_0}p_1^{a_1}...p_r^{a_r}-1$.



      Think about what the totient function entails: the number of integers less than the input that are coprime with it. Only with prime inputs is this one less than the input.



      The totient function, however, does retain two properties, namely as reuns mentioned, that $phi(p^k)=p^k-p^{k-1}$ and $phi(n)phi(m)=phi(mn)$ when $m$ and $n$ are coprime. The proof of these is easily verifiable given a google search.



      Thus, we can have a "direct" approach. Since powers of primes are always coprime to one another,
      begin{align*}
      phi(n)&=phi(p_0^{a_0}p_1^{a_1}...p_r^{a_r})\
      &=phi(p_0^{a_0})phi(p_1^{a_1})...phi(p_r^{a_r})\
      &=(p_0^{a_0}-p_0^{a_0-1})(p_1^{a_1}-p_1^{a_1-1})...(p_r^{a_r}-p_r^{a_r-1})\
      &=p_0^{a_0}(1-1/p_0)p_1^{a_1}(1-1/p_1)...p_r^{a_r}(1-1/p_r)\
      &=ndisplaystyleprod_{p|n}(1-1/p).
      end{align*}






      share|cite|improve this answer









      $endgroup$
















        1












        1








        1





        $begingroup$

        To be precise, your "kind" of derivation is not possible. This is because $phi(p_0^{a_0}p_1^{a_1}...p_r^{a_r})neq p_0^{a_0}p_1^{a_1}...p_r^{a_r}-1$.



        Think about what the totient function entails: the number of integers less than the input that are coprime with it. Only with prime inputs is this one less than the input.



        The totient function, however, does retain two properties, namely as reuns mentioned, that $phi(p^k)=p^k-p^{k-1}$ and $phi(n)phi(m)=phi(mn)$ when $m$ and $n$ are coprime. The proof of these is easily verifiable given a google search.



        Thus, we can have a "direct" approach. Since powers of primes are always coprime to one another,
        begin{align*}
        phi(n)&=phi(p_0^{a_0}p_1^{a_1}...p_r^{a_r})\
        &=phi(p_0^{a_0})phi(p_1^{a_1})...phi(p_r^{a_r})\
        &=(p_0^{a_0}-p_0^{a_0-1})(p_1^{a_1}-p_1^{a_1-1})...(p_r^{a_r}-p_r^{a_r-1})\
        &=p_0^{a_0}(1-1/p_0)p_1^{a_1}(1-1/p_1)...p_r^{a_r}(1-1/p_r)\
        &=ndisplaystyleprod_{p|n}(1-1/p).
        end{align*}






        share|cite|improve this answer









        $endgroup$



        To be precise, your "kind" of derivation is not possible. This is because $phi(p_0^{a_0}p_1^{a_1}...p_r^{a_r})neq p_0^{a_0}p_1^{a_1}...p_r^{a_r}-1$.



        Think about what the totient function entails: the number of integers less than the input that are coprime with it. Only with prime inputs is this one less than the input.



        The totient function, however, does retain two properties, namely as reuns mentioned, that $phi(p^k)=p^k-p^{k-1}$ and $phi(n)phi(m)=phi(mn)$ when $m$ and $n$ are coprime. The proof of these is easily verifiable given a google search.



        Thus, we can have a "direct" approach. Since powers of primes are always coprime to one another,
        begin{align*}
        phi(n)&=phi(p_0^{a_0}p_1^{a_1}...p_r^{a_r})\
        &=phi(p_0^{a_0})phi(p_1^{a_1})...phi(p_r^{a_r})\
        &=(p_0^{a_0}-p_0^{a_0-1})(p_1^{a_1}-p_1^{a_1-1})...(p_r^{a_r}-p_r^{a_r-1})\
        &=p_0^{a_0}(1-1/p_0)p_1^{a_1}(1-1/p_1)...p_r^{a_r}(1-1/p_r)\
        &=ndisplaystyleprod_{p|n}(1-1/p).
        end{align*}







        share|cite|improve this answer












        share|cite|improve this answer



        share|cite|improve this answer










        answered Jan 15 at 20:50









        Tejas RaoTejas Rao

        16611




        16611






























            draft saved

            draft discarded




















































            Thanks for contributing an answer to Mathematics Stack Exchange!


            • Please be sure to answer the question. Provide details and share your research!

            But avoid



            • Asking for help, clarification, or responding to other answers.

            • Making statements based on opinion; back them up with references or personal experience.


            Use MathJax to format equations. MathJax reference.


            To learn more, see our tips on writing great answers.




            draft saved


            draft discarded














            StackExchange.ready(
            function () {
            StackExchange.openid.initPostLogin('.new-post-login', 'https%3a%2f%2fmath.stackexchange.com%2fquestions%2f3074184%2fdirect-passage-from-n-prime-to-n-non-prime-in-euler-totient-function-phi%23new-answer', 'question_page');
            }
            );

            Post as a guest















            Required, but never shown





















































            Required, but never shown














            Required, but never shown












            Required, but never shown







            Required, but never shown

































            Required, but never shown














            Required, but never shown












            Required, but never shown







            Required, but never shown







            Popular posts from this blog

            Mario Kart Wii

            What does “Dominus providebit” mean?

            Antonio Litta Visconti Arese